LSAT and Law School Admissions Forum

Get expert LSAT preparation and law school admissions advice from PowerScore Test Preparation.

 Administrator
PowerScore Staff
  • PowerScore Staff
  • Posts: 8917
  • Joined: Feb 02, 2011
|
#40629
Complete Question Explanation
(The complete setup for this game can be found here: lsat/viewtopic.php?t=14136)

The correct answer choice is (C)

If I is the third school to which juices are delivered, then from the inference resulting from the fourth rule, H must be the first school to which juices are delivered (and H also must be the fourth school to which snacks are delivered):

PT69_Game_#3_#14_diagram 1.png
From the second rule, G cannot be the fourth school to which juices are delivered, and so G must be the second school to which juices are delivered, leaving F to be the fourth school to which juices are delivered:

PT69_Game_#3_#14_diagram 2.png
The only uncertainty in the game, then, is the order of the F and I snack deliveries. Thus, in a Could Be True question such as this one, you should immediately seek answer choices that contain the F or I (and preferably both since each answer choice references two schools) snacks deliveries. Only answer choices (C) and (D) qualify, and answer choice (C) is superior because it contains both F and I. Because the snacks could be delivered to F at some time before they are delivered to I, answer choice (C) could be true and is therefore correct.
You do not have the required permissions to view the files attached to this post.

Get the most out of your LSAT Prep Plus subscription.

Analyze and track your performance with our Testing and Analytics Package.